Can I define a measurable function using base of a topology?












0












$begingroup$


We know the more general definition of a measurable funnction: let $(X,mathcal{X})$ and $(Y,tau)$ a measurable and topological space, respectively. The function $f:Xto Y$ is a $mathcal{X}$-measurable function if $f^{-1}(V) in mathcal{X}$, $forall V in tau$.



I would like to know if it is true that we can define the measurability of the function $f$ using a base of the topology $tau$. That is, suposse that $beta$ is a base of $tau$. Then, I wold like to prove that



$$f hbox{ is } mathcal{X}hbox{-measurable function} Leftrightarrow f^{-1}(B) in mathcal{X},quad forall B in beta $$



We know that any $V in tau$ can be written as



$$V = bigcup_{A in mathcal{g}} A,quad mathcal{g} subset beta$$



with $g$ not necessarily enumerable. One implication of my goal is trivial. Any element of $beta$ is open, that is, it is a element of $tau$. Then, if $f$ is $mathcal{X}$-measurable function, then $f^{-1}(B) in mathcal{X}, forall B in beta$. But, because $g in beta$ is not necessarily enumerable, I can not conclude the other implication. That is, I can not conclude that



$$f^{-1}(V) = bigcup_{A in mathcal{g}} f^{-1}(A) in mathcal{X}$$



I would like to know if it is possible to circumvent this or in what cases this result is valid.










share|cite|improve this question









$endgroup$












  • $begingroup$
    If the $Y$ is second countable the any open set is a countable union of sets from the base, so you get measurability. In general you cannot conclude that $f$ is measurable.
    $endgroup$
    – Kavi Rama Murthy
    Dec 18 '18 at 5:31










  • $begingroup$
    Tell me more about this property. Are metric spaces second countable?
    $endgroup$
    – orrillo
    Dec 18 '18 at 5:48






  • 1




    $begingroup$
    A metric space is second countable iff it is separable (in the sense there is a countable subset which is dense). A standard example of a metric space which is not separable is $ell ^{infty}$ the space of all bounded sequences of real numbers with the metric $d((a_n),(b_n))=sup{|a_n-b_n|:ngeq 1}$.
    $endgroup$
    – Kavi Rama Murthy
    Dec 18 '18 at 5:52










  • $begingroup$
    $mathbb{R}$ is separable because $mathbb{Q}$ is dense in $mathbb{R}$. Then $mathbb{R}$ has a enumerable base. Can I conclude that ${(q_1,q_2) , q_1 < q_2 , q_i in mathbb{Q}}$ is a base of the topology of $mathbb{R}$?
    $endgroup$
    – orrillo
    Dec 18 '18 at 6:04








  • 1




    $begingroup$
    All Euclidean spaces are separable (hence second countable) so your definition of measurability works. Intervals with rational end points form a countable base in $mathbb R$, rectangles with rational coordinatesform a countable base in $mathbb R^{2}$, etc.
    $endgroup$
    – Kavi Rama Murthy
    Dec 18 '18 at 6:08


















0












$begingroup$


We know the more general definition of a measurable funnction: let $(X,mathcal{X})$ and $(Y,tau)$ a measurable and topological space, respectively. The function $f:Xto Y$ is a $mathcal{X}$-measurable function if $f^{-1}(V) in mathcal{X}$, $forall V in tau$.



I would like to know if it is true that we can define the measurability of the function $f$ using a base of the topology $tau$. That is, suposse that $beta$ is a base of $tau$. Then, I wold like to prove that



$$f hbox{ is } mathcal{X}hbox{-measurable function} Leftrightarrow f^{-1}(B) in mathcal{X},quad forall B in beta $$



We know that any $V in tau$ can be written as



$$V = bigcup_{A in mathcal{g}} A,quad mathcal{g} subset beta$$



with $g$ not necessarily enumerable. One implication of my goal is trivial. Any element of $beta$ is open, that is, it is a element of $tau$. Then, if $f$ is $mathcal{X}$-measurable function, then $f^{-1}(B) in mathcal{X}, forall B in beta$. But, because $g in beta$ is not necessarily enumerable, I can not conclude the other implication. That is, I can not conclude that



$$f^{-1}(V) = bigcup_{A in mathcal{g}} f^{-1}(A) in mathcal{X}$$



I would like to know if it is possible to circumvent this or in what cases this result is valid.










share|cite|improve this question









$endgroup$












  • $begingroup$
    If the $Y$ is second countable the any open set is a countable union of sets from the base, so you get measurability. In general you cannot conclude that $f$ is measurable.
    $endgroup$
    – Kavi Rama Murthy
    Dec 18 '18 at 5:31










  • $begingroup$
    Tell me more about this property. Are metric spaces second countable?
    $endgroup$
    – orrillo
    Dec 18 '18 at 5:48






  • 1




    $begingroup$
    A metric space is second countable iff it is separable (in the sense there is a countable subset which is dense). A standard example of a metric space which is not separable is $ell ^{infty}$ the space of all bounded sequences of real numbers with the metric $d((a_n),(b_n))=sup{|a_n-b_n|:ngeq 1}$.
    $endgroup$
    – Kavi Rama Murthy
    Dec 18 '18 at 5:52










  • $begingroup$
    $mathbb{R}$ is separable because $mathbb{Q}$ is dense in $mathbb{R}$. Then $mathbb{R}$ has a enumerable base. Can I conclude that ${(q_1,q_2) , q_1 < q_2 , q_i in mathbb{Q}}$ is a base of the topology of $mathbb{R}$?
    $endgroup$
    – orrillo
    Dec 18 '18 at 6:04








  • 1




    $begingroup$
    All Euclidean spaces are separable (hence second countable) so your definition of measurability works. Intervals with rational end points form a countable base in $mathbb R$, rectangles with rational coordinatesform a countable base in $mathbb R^{2}$, etc.
    $endgroup$
    – Kavi Rama Murthy
    Dec 18 '18 at 6:08
















0












0








0





$begingroup$


We know the more general definition of a measurable funnction: let $(X,mathcal{X})$ and $(Y,tau)$ a measurable and topological space, respectively. The function $f:Xto Y$ is a $mathcal{X}$-measurable function if $f^{-1}(V) in mathcal{X}$, $forall V in tau$.



I would like to know if it is true that we can define the measurability of the function $f$ using a base of the topology $tau$. That is, suposse that $beta$ is a base of $tau$. Then, I wold like to prove that



$$f hbox{ is } mathcal{X}hbox{-measurable function} Leftrightarrow f^{-1}(B) in mathcal{X},quad forall B in beta $$



We know that any $V in tau$ can be written as



$$V = bigcup_{A in mathcal{g}} A,quad mathcal{g} subset beta$$



with $g$ not necessarily enumerable. One implication of my goal is trivial. Any element of $beta$ is open, that is, it is a element of $tau$. Then, if $f$ is $mathcal{X}$-measurable function, then $f^{-1}(B) in mathcal{X}, forall B in beta$. But, because $g in beta$ is not necessarily enumerable, I can not conclude the other implication. That is, I can not conclude that



$$f^{-1}(V) = bigcup_{A in mathcal{g}} f^{-1}(A) in mathcal{X}$$



I would like to know if it is possible to circumvent this or in what cases this result is valid.










share|cite|improve this question









$endgroup$




We know the more general definition of a measurable funnction: let $(X,mathcal{X})$ and $(Y,tau)$ a measurable and topological space, respectively. The function $f:Xto Y$ is a $mathcal{X}$-measurable function if $f^{-1}(V) in mathcal{X}$, $forall V in tau$.



I would like to know if it is true that we can define the measurability of the function $f$ using a base of the topology $tau$. That is, suposse that $beta$ is a base of $tau$. Then, I wold like to prove that



$$f hbox{ is } mathcal{X}hbox{-measurable function} Leftrightarrow f^{-1}(B) in mathcal{X},quad forall B in beta $$



We know that any $V in tau$ can be written as



$$V = bigcup_{A in mathcal{g}} A,quad mathcal{g} subset beta$$



with $g$ not necessarily enumerable. One implication of my goal is trivial. Any element of $beta$ is open, that is, it is a element of $tau$. Then, if $f$ is $mathcal{X}$-measurable function, then $f^{-1}(B) in mathcal{X}, forall B in beta$. But, because $g in beta$ is not necessarily enumerable, I can not conclude the other implication. That is, I can not conclude that



$$f^{-1}(V) = bigcup_{A in mathcal{g}} f^{-1}(A) in mathcal{X}$$



I would like to know if it is possible to circumvent this or in what cases this result is valid.







general-topology measure-theory






share|cite|improve this question













share|cite|improve this question











share|cite|improve this question




share|cite|improve this question










asked Dec 18 '18 at 5:20









orrilloorrillo

298110




298110












  • $begingroup$
    If the $Y$ is second countable the any open set is a countable union of sets from the base, so you get measurability. In general you cannot conclude that $f$ is measurable.
    $endgroup$
    – Kavi Rama Murthy
    Dec 18 '18 at 5:31










  • $begingroup$
    Tell me more about this property. Are metric spaces second countable?
    $endgroup$
    – orrillo
    Dec 18 '18 at 5:48






  • 1




    $begingroup$
    A metric space is second countable iff it is separable (in the sense there is a countable subset which is dense). A standard example of a metric space which is not separable is $ell ^{infty}$ the space of all bounded sequences of real numbers with the metric $d((a_n),(b_n))=sup{|a_n-b_n|:ngeq 1}$.
    $endgroup$
    – Kavi Rama Murthy
    Dec 18 '18 at 5:52










  • $begingroup$
    $mathbb{R}$ is separable because $mathbb{Q}$ is dense in $mathbb{R}$. Then $mathbb{R}$ has a enumerable base. Can I conclude that ${(q_1,q_2) , q_1 < q_2 , q_i in mathbb{Q}}$ is a base of the topology of $mathbb{R}$?
    $endgroup$
    – orrillo
    Dec 18 '18 at 6:04








  • 1




    $begingroup$
    All Euclidean spaces are separable (hence second countable) so your definition of measurability works. Intervals with rational end points form a countable base in $mathbb R$, rectangles with rational coordinatesform a countable base in $mathbb R^{2}$, etc.
    $endgroup$
    – Kavi Rama Murthy
    Dec 18 '18 at 6:08




















  • $begingroup$
    If the $Y$ is second countable the any open set is a countable union of sets from the base, so you get measurability. In general you cannot conclude that $f$ is measurable.
    $endgroup$
    – Kavi Rama Murthy
    Dec 18 '18 at 5:31










  • $begingroup$
    Tell me more about this property. Are metric spaces second countable?
    $endgroup$
    – orrillo
    Dec 18 '18 at 5:48






  • 1




    $begingroup$
    A metric space is second countable iff it is separable (in the sense there is a countable subset which is dense). A standard example of a metric space which is not separable is $ell ^{infty}$ the space of all bounded sequences of real numbers with the metric $d((a_n),(b_n))=sup{|a_n-b_n|:ngeq 1}$.
    $endgroup$
    – Kavi Rama Murthy
    Dec 18 '18 at 5:52










  • $begingroup$
    $mathbb{R}$ is separable because $mathbb{Q}$ is dense in $mathbb{R}$. Then $mathbb{R}$ has a enumerable base. Can I conclude that ${(q_1,q_2) , q_1 < q_2 , q_i in mathbb{Q}}$ is a base of the topology of $mathbb{R}$?
    $endgroup$
    – orrillo
    Dec 18 '18 at 6:04








  • 1




    $begingroup$
    All Euclidean spaces are separable (hence second countable) so your definition of measurability works. Intervals with rational end points form a countable base in $mathbb R$, rectangles with rational coordinatesform a countable base in $mathbb R^{2}$, etc.
    $endgroup$
    – Kavi Rama Murthy
    Dec 18 '18 at 6:08


















$begingroup$
If the $Y$ is second countable the any open set is a countable union of sets from the base, so you get measurability. In general you cannot conclude that $f$ is measurable.
$endgroup$
– Kavi Rama Murthy
Dec 18 '18 at 5:31




$begingroup$
If the $Y$ is second countable the any open set is a countable union of sets from the base, so you get measurability. In general you cannot conclude that $f$ is measurable.
$endgroup$
– Kavi Rama Murthy
Dec 18 '18 at 5:31












$begingroup$
Tell me more about this property. Are metric spaces second countable?
$endgroup$
– orrillo
Dec 18 '18 at 5:48




$begingroup$
Tell me more about this property. Are metric spaces second countable?
$endgroup$
– orrillo
Dec 18 '18 at 5:48




1




1




$begingroup$
A metric space is second countable iff it is separable (in the sense there is a countable subset which is dense). A standard example of a metric space which is not separable is $ell ^{infty}$ the space of all bounded sequences of real numbers with the metric $d((a_n),(b_n))=sup{|a_n-b_n|:ngeq 1}$.
$endgroup$
– Kavi Rama Murthy
Dec 18 '18 at 5:52




$begingroup$
A metric space is second countable iff it is separable (in the sense there is a countable subset which is dense). A standard example of a metric space which is not separable is $ell ^{infty}$ the space of all bounded sequences of real numbers with the metric $d((a_n),(b_n))=sup{|a_n-b_n|:ngeq 1}$.
$endgroup$
– Kavi Rama Murthy
Dec 18 '18 at 5:52












$begingroup$
$mathbb{R}$ is separable because $mathbb{Q}$ is dense in $mathbb{R}$. Then $mathbb{R}$ has a enumerable base. Can I conclude that ${(q_1,q_2) , q_1 < q_2 , q_i in mathbb{Q}}$ is a base of the topology of $mathbb{R}$?
$endgroup$
– orrillo
Dec 18 '18 at 6:04






$begingroup$
$mathbb{R}$ is separable because $mathbb{Q}$ is dense in $mathbb{R}$. Then $mathbb{R}$ has a enumerable base. Can I conclude that ${(q_1,q_2) , q_1 < q_2 , q_i in mathbb{Q}}$ is a base of the topology of $mathbb{R}$?
$endgroup$
– orrillo
Dec 18 '18 at 6:04






1




1




$begingroup$
All Euclidean spaces are separable (hence second countable) so your definition of measurability works. Intervals with rational end points form a countable base in $mathbb R$, rectangles with rational coordinatesform a countable base in $mathbb R^{2}$, etc.
$endgroup$
– Kavi Rama Murthy
Dec 18 '18 at 6:08






$begingroup$
All Euclidean spaces are separable (hence second countable) so your definition of measurability works. Intervals with rational end points form a countable base in $mathbb R$, rectangles with rational coordinatesform a countable base in $mathbb R^{2}$, etc.
$endgroup$
– Kavi Rama Murthy
Dec 18 '18 at 6:08












1 Answer
1






active

oldest

votes


















1












$begingroup$

This is too long for the comment section.



You are probably after the following result. On any set $mathrm{Z},$ denote by $sigma(mathscr{A})$ the minimal sigma algebra containing the subset $mathscr{A}$ of the power set of $mathrm{Z}$. If $varphi$ is a function with values in $mathrm{Z},$ we also denote by $varphi(mathscr{A})$ the set of $varphi^{-1}(mathrm{A})$ as $mathrm{A}$ runs through $mathscr{A}.$



Proposition. Let $f$ be any function $mathrm{X} to mathrm{Y}$ and let $mathscr{Y}$ be any subset of the power set of $mathrm{Y}.$ Then, $f^{-1}(sigma(mathscr{Y})) = sigma(f^{-1}(mathscr{Y})).$



Proof. You probably already know that the set of preimages of a sigma algebra conforms a sigma algebra, this proves at once the $supset.$ For the other side, consider the set $mathscr{Y}'$ of sets $mathrm{E}$ in $sigma(mathscr{Y})$ such that $f^{-1}(mathrm{E}) in sigma(f^{-1}(mathscr{Y})).$ Straightforwardly, $mathscr{Y}'$ is a sigma field containing $mathscr{Y},$ hence, it also contains $mathscr{Y},$ this proves $subset.$ Q.E.D.






share|cite|improve this answer









$endgroup$













    Your Answer





    StackExchange.ifUsing("editor", function () {
    return StackExchange.using("mathjaxEditing", function () {
    StackExchange.MarkdownEditor.creationCallbacks.add(function (editor, postfix) {
    StackExchange.mathjaxEditing.prepareWmdForMathJax(editor, postfix, [["$", "$"], ["\\(","\\)"]]);
    });
    });
    }, "mathjax-editing");

    StackExchange.ready(function() {
    var channelOptions = {
    tags: "".split(" "),
    id: "69"
    };
    initTagRenderer("".split(" "), "".split(" "), channelOptions);

    StackExchange.using("externalEditor", function() {
    // Have to fire editor after snippets, if snippets enabled
    if (StackExchange.settings.snippets.snippetsEnabled) {
    StackExchange.using("snippets", function() {
    createEditor();
    });
    }
    else {
    createEditor();
    }
    });

    function createEditor() {
    StackExchange.prepareEditor({
    heartbeatType: 'answer',
    autoActivateHeartbeat: false,
    convertImagesToLinks: true,
    noModals: true,
    showLowRepImageUploadWarning: true,
    reputationToPostImages: 10,
    bindNavPrevention: true,
    postfix: "",
    imageUploader: {
    brandingHtml: "Powered by u003ca class="icon-imgur-white" href="https://imgur.com/"u003eu003c/au003e",
    contentPolicyHtml: "User contributions licensed under u003ca href="https://creativecommons.org/licenses/by-sa/3.0/"u003ecc by-sa 3.0 with attribution requiredu003c/au003e u003ca href="https://stackoverflow.com/legal/content-policy"u003e(content policy)u003c/au003e",
    allowUrls: true
    },
    noCode: true, onDemand: true,
    discardSelector: ".discard-answer"
    ,immediatelyShowMarkdownHelp:true
    });


    }
    });














    draft saved

    draft discarded


















    StackExchange.ready(
    function () {
    StackExchange.openid.initPostLogin('.new-post-login', 'https%3a%2f%2fmath.stackexchange.com%2fquestions%2f3044807%2fcan-i-define-a-measurable-function-using-base-of-a-topology%23new-answer', 'question_page');
    }
    );

    Post as a guest















    Required, but never shown

























    1 Answer
    1






    active

    oldest

    votes








    1 Answer
    1






    active

    oldest

    votes









    active

    oldest

    votes






    active

    oldest

    votes









    1












    $begingroup$

    This is too long for the comment section.



    You are probably after the following result. On any set $mathrm{Z},$ denote by $sigma(mathscr{A})$ the minimal sigma algebra containing the subset $mathscr{A}$ of the power set of $mathrm{Z}$. If $varphi$ is a function with values in $mathrm{Z},$ we also denote by $varphi(mathscr{A})$ the set of $varphi^{-1}(mathrm{A})$ as $mathrm{A}$ runs through $mathscr{A}.$



    Proposition. Let $f$ be any function $mathrm{X} to mathrm{Y}$ and let $mathscr{Y}$ be any subset of the power set of $mathrm{Y}.$ Then, $f^{-1}(sigma(mathscr{Y})) = sigma(f^{-1}(mathscr{Y})).$



    Proof. You probably already know that the set of preimages of a sigma algebra conforms a sigma algebra, this proves at once the $supset.$ For the other side, consider the set $mathscr{Y}'$ of sets $mathrm{E}$ in $sigma(mathscr{Y})$ such that $f^{-1}(mathrm{E}) in sigma(f^{-1}(mathscr{Y})).$ Straightforwardly, $mathscr{Y}'$ is a sigma field containing $mathscr{Y},$ hence, it also contains $mathscr{Y},$ this proves $subset.$ Q.E.D.






    share|cite|improve this answer









    $endgroup$


















      1












      $begingroup$

      This is too long for the comment section.



      You are probably after the following result. On any set $mathrm{Z},$ denote by $sigma(mathscr{A})$ the minimal sigma algebra containing the subset $mathscr{A}$ of the power set of $mathrm{Z}$. If $varphi$ is a function with values in $mathrm{Z},$ we also denote by $varphi(mathscr{A})$ the set of $varphi^{-1}(mathrm{A})$ as $mathrm{A}$ runs through $mathscr{A}.$



      Proposition. Let $f$ be any function $mathrm{X} to mathrm{Y}$ and let $mathscr{Y}$ be any subset of the power set of $mathrm{Y}.$ Then, $f^{-1}(sigma(mathscr{Y})) = sigma(f^{-1}(mathscr{Y})).$



      Proof. You probably already know that the set of preimages of a sigma algebra conforms a sigma algebra, this proves at once the $supset.$ For the other side, consider the set $mathscr{Y}'$ of sets $mathrm{E}$ in $sigma(mathscr{Y})$ such that $f^{-1}(mathrm{E}) in sigma(f^{-1}(mathscr{Y})).$ Straightforwardly, $mathscr{Y}'$ is a sigma field containing $mathscr{Y},$ hence, it also contains $mathscr{Y},$ this proves $subset.$ Q.E.D.






      share|cite|improve this answer









      $endgroup$
















        1












        1








        1





        $begingroup$

        This is too long for the comment section.



        You are probably after the following result. On any set $mathrm{Z},$ denote by $sigma(mathscr{A})$ the minimal sigma algebra containing the subset $mathscr{A}$ of the power set of $mathrm{Z}$. If $varphi$ is a function with values in $mathrm{Z},$ we also denote by $varphi(mathscr{A})$ the set of $varphi^{-1}(mathrm{A})$ as $mathrm{A}$ runs through $mathscr{A}.$



        Proposition. Let $f$ be any function $mathrm{X} to mathrm{Y}$ and let $mathscr{Y}$ be any subset of the power set of $mathrm{Y}.$ Then, $f^{-1}(sigma(mathscr{Y})) = sigma(f^{-1}(mathscr{Y})).$



        Proof. You probably already know that the set of preimages of a sigma algebra conforms a sigma algebra, this proves at once the $supset.$ For the other side, consider the set $mathscr{Y}'$ of sets $mathrm{E}$ in $sigma(mathscr{Y})$ such that $f^{-1}(mathrm{E}) in sigma(f^{-1}(mathscr{Y})).$ Straightforwardly, $mathscr{Y}'$ is a sigma field containing $mathscr{Y},$ hence, it also contains $mathscr{Y},$ this proves $subset.$ Q.E.D.






        share|cite|improve this answer









        $endgroup$



        This is too long for the comment section.



        You are probably after the following result. On any set $mathrm{Z},$ denote by $sigma(mathscr{A})$ the minimal sigma algebra containing the subset $mathscr{A}$ of the power set of $mathrm{Z}$. If $varphi$ is a function with values in $mathrm{Z},$ we also denote by $varphi(mathscr{A})$ the set of $varphi^{-1}(mathrm{A})$ as $mathrm{A}$ runs through $mathscr{A}.$



        Proposition. Let $f$ be any function $mathrm{X} to mathrm{Y}$ and let $mathscr{Y}$ be any subset of the power set of $mathrm{Y}.$ Then, $f^{-1}(sigma(mathscr{Y})) = sigma(f^{-1}(mathscr{Y})).$



        Proof. You probably already know that the set of preimages of a sigma algebra conforms a sigma algebra, this proves at once the $supset.$ For the other side, consider the set $mathscr{Y}'$ of sets $mathrm{E}$ in $sigma(mathscr{Y})$ such that $f^{-1}(mathrm{E}) in sigma(f^{-1}(mathscr{Y})).$ Straightforwardly, $mathscr{Y}'$ is a sigma field containing $mathscr{Y},$ hence, it also contains $mathscr{Y},$ this proves $subset.$ Q.E.D.







        share|cite|improve this answer












        share|cite|improve this answer



        share|cite|improve this answer










        answered Dec 18 '18 at 7:00









        Will M.Will M.

        2,715315




        2,715315






























            draft saved

            draft discarded




















































            Thanks for contributing an answer to Mathematics Stack Exchange!


            • Please be sure to answer the question. Provide details and share your research!

            But avoid



            • Asking for help, clarification, or responding to other answers.

            • Making statements based on opinion; back them up with references or personal experience.


            Use MathJax to format equations. MathJax reference.


            To learn more, see our tips on writing great answers.




            draft saved


            draft discarded














            StackExchange.ready(
            function () {
            StackExchange.openid.initPostLogin('.new-post-login', 'https%3a%2f%2fmath.stackexchange.com%2fquestions%2f3044807%2fcan-i-define-a-measurable-function-using-base-of-a-topology%23new-answer', 'question_page');
            }
            );

            Post as a guest















            Required, but never shown





















































            Required, but never shown














            Required, but never shown












            Required, but never shown







            Required, but never shown

































            Required, but never shown














            Required, but never shown












            Required, but never shown







            Required, but never shown







            Popular posts from this blog

            To store a contact into the json file from server.js file using a class in NodeJS

            Redirect URL with Chrome Remote Debugging Android Devices

            Dieringhausen